Which one of the following pairs of employees is such that at least one member of the pair volunteers?

Vennela-Vellanki on August 18, 2020

Why is E wrong?

Why is E wrong in this question?

Reply
Create a free account to read and take part in forum discussions.

Already have an account? log in

shunhe on August 19, 2020

Hi @Vennela-Vellanki,

Thanks for the question! So this question is asking us for a pair of employees such that at least one member of the pair volunteers. So the correct answer is going to be one in which we take both of the people out, and we can’t put together a successful game, one in which we necessarily run into some contradiction.

So let’s take a look at (E), which suggests S and T. OK, so let’s say that both S and T aren’t in the game. Can the game still work? If it can, then it’s not a pair where at least one of them have to volunteer. Well, if S and T are out, then we know that M is out by the contrapositive of rule 2, R is out by the contrapositive of rule 1. Who’s in? Well, L is in from rule 4, and V is in from rule 3. Is there a contradiction here? No, none of the rules are violated, so we can take S and T out with no problems. And that means that we don’t need at least one of the pair, so (E) is incorrect.

Hope this helps! Feel free to ask any other questions that you might have.